Trạng thái giới hạn mạch dưới cho mạch độ sâu giới hạn polylog


17

AC0pAC0[q]AC0[q]qgcd(p,q)=1. Tuy nhiên, việc thu được kết quả giới hạn cụ thể trên các mạch độ sâu đa giác dường như nằm ngoài tầm với bằng cách sử dụng các phương pháp cổ điển như hạn chế đầu vào và xấp xỉ đa thức trên các trường hữu hạn.

Tôi biết một bài báo STOC'96 dẫn đến lý thuyết phức tạp hình học và cho thấy rằng tính toán song song hiệu quả bằng cách sử dụng các hoạt động mà không có bit-khôn ngoan có thể tính toán được vấn đề dòng chảy chi phí tối thiểu.

Điều này có nghĩa là trong một số cài đặt giới hạn nhất định, chúng tôi có thể chứng minh giới hạn thấp hơn đối với một số vấn đề -complete.PNCP

Đầu tiên, có phương pháp hoặc kỹ thuật nào khác có thể là phương pháp hợp lý để chứng minh giới hạn độ sâu đa mạch không?

Thứ hai, tuyên bố sau đây hữu ích như thế nào đối với cộng đồng lý thuyết?

Kích thước của một mạch tính toán hàm Boolean ít nhất là , trong đó là một số lượng toán học tùy thuộc vào độ cứng của hàm mục tiêu . Ví dụ, giá trị của có thể là đại lượng tổ hợp như sai lệch, đại lượng tuyến tính như thứ hạng của một loại ma trận nhất định trên một trường hoặc một số lượng hoàn toàn mới chưa từng được sử dụng trong lý thuyết phức tạp.f : { 0 , 1 } n{ 0 , 1 } l l f lNCf:{0,1}n{0,1}llfl


6
Một lời cảnh báo là theo thứ tự: thậm chí độ sâu logarit nếu không được hiểu rõ. Chúng tôi vẫn không có giới hạn siêu tuyến tính (!) Cho NC ^ 1-mạch. Ở đây, độ cứng của ma trận là một "đại lượng tổ hợp" mong muốn, nhưng chúng ta thiếu các giới hạn dưới đủ mạnh về đại lượng này. Thậm chí đáng buồn hơn, không có giới hạn dưới siêu tuyến tính nào được biết đến với NC ^ 1 mạch điện toán biến đổi tuyến tính f (x) = Axe trên GF (2), ngay cả khi chỉ cho phép các fanin-2 XOR làm cổng. (Hầu hết tất cả các ma trận A sau đó yêu cầu khoảng n ^ 2 / \ log n cổng, ở bất kỳ độ sâu nào.)
Stasys

@Stasys, tôi nghĩ nhận xét của bạn có thể là một câu trả lời.
Kaveh

Câu trả lời:


16

Về các kỹ thuật để chứng minh các giới hạn dưới sâu của mạch poly-log, tất cả các phương pháp hiện tại đều hoạt động trong các cài đặt hạn chế . Giống như, trong công việc dẫn đến GCT mà bạn đề cập, giới hạn dưới áp dụng cho mô hình PRAM bị hạn chế mà không có thao tác bit.

Trong một hạn chế khác, đó là hạn chế đơn điệu đối với các hàm boolean đơn điệu, có một cách tiếp cận Fourier-phân tích (hoặc liệt kê kết hợp) để chứng minh các giới hạn thấp hơn mạch đơn, trong công việc chung của tôi với Aaron Potechin ( ECCCSTOC ). Điều này cải thiện kết quả trước đó của Ran Raz và Pierre McKenzie, người mở rộng khung trò chơi giao tiếp của Mauricio Karchmer và Avi Wigderson liên quan đến độ sâu mạch.

Một dòng nghiên cứu khác để mở rộng trò chơi KarchmerTHER Wigderson đã được đề xuất như một trò chơi giao tiếp được giới thiệu bởi Scott Aaronson và Avi Wigderson, người mở rộng giao thức cạnh tranh được đề xuất như một cách tiếp cận để tách NC khỏi P bởi Gillat Kol và Ran Raz ( ECCCITCS ).

Ngoài việc nghiên cứu hạn chế cú pháp của tính đơn điệu, còn có một cách tiếp cận để nghiên cứu một hạn chế ngữ nghĩa liên quan đến các trò chơi cuội (được gọi là các chương trình phân nhánh tiết kiệm) của Stephen Cook, Pierre McKenzie, Dustin Wehr, Mark Braverman và Rahul Santhanam. Có một giới hạn dưới mạnh mẽ dưới sự hạn chế tiết kiệm của Dustin Wehr, phù hợp với giới hạn trên được biết đến nhiều nhất cho các vấn đề P-Complete. Những kết quả liên quan đến độ phức tạp không gian xác định, giới hạn mà thấp hơn thời gian song song hoặc mạch sâu bởi kết quả mô phỏng được biết đến (ví dụ như từ ).AlternatingTime[t]DeterministicSpace[t]

Về câu hỏi liên quan đến kích thước và độ sâu của mạch, cách tiếp cận sau đây có thể liên quan. Richard Lipton và Ryan Williams cho thấy rằng, cho một đủ mạnh thấp hơn bị ràng buộc vào chiều sâu (tức là ), một kích thước yếu thấp hơn ràng buộc (tức là n 1 + Ω ( 1 ) ) có thể tách NC từ P. Kết quả này theo sau một đối số đánh đổi chiều sâu dựa trên các mô phỏng tôn trọng khối. Một kết quả trước đó về độ sâu giao dịch cho kích thước là do Allender và Koucký dựa trên ý tưởng về khả năng tự giảm, nhưng nó đã nghiên cứu các lớp phức tạp nhỏ hơn như NC 1 và NL.n1O(1)n1+Ω(1)1

Lưu ý rằng trong số các cách tiếp cận được đề cập ở trên, một số trong số chúng xem xét cả kích thước và độ sâu của mạch, trong khi các cách tiếp cận khác chỉ xem xét độ sâu của mạch. Cụ thể, cách tiếp cận bán hình học của Mulmuley , cách tiếp cận giao thức cạnh tranh được nghiên cứu bởi KolTHER Raz , và cách tiếp cận đánh đổi chiều sâu của Allender tựa KouckýLipton, Williams đều quan tâm đến cả kích thước và độ sâu của các mạch. Các kết quả trong ChanTHER Potechin , RazTHER McKenzie , CookTHER McKenzieifer WehrTHER BravermanTHER SanthanamWehr đưa ra các giới hạn thấp hơn về mạch dưới các cài đặt hạn chế bất kể kích thước. Ngoài ra, trò chơi giao tiếp được giới thiệu củaAaronsonTHER Wigderson chỉ quan tâm đến độ sâu mạch.

Vẫn còn phù hợp với kiến ​​thức của chúng tôi rằng một số vấn đề hoàn thành P không thể được tính bằng các mạch có độ sâu nhỏ (tức là ), bất kể kích thước. Nếu kích thước không quan trọng đối với các mạch có độ sâu nhỏ (của quạt có giới hạn), thì có lẽ nên tập trung nhiều hơn vào độ sâu của mạch, hơn là tập trung vào kích thước của các mạch có độ sâu nhỏ.logO(1)n


cảm ơn! Như bạn đã biết, một tuyên bố trong quý 2 không phải ai cũng tìm thấy, phải không? Đó là, không giống như các phương thức liên kết giới hạn độ phức tạp thấp hơn, chúng ta không có bất kỳ số lượng toán học nào cho các giới hạn thấp hơn của mạch NC?
shen

@shen, tôi đã thêm hai đoạn văn ở cuối. Hy vọng rằng nó hữu ích.
siuman

2
Ý tưởng rằng các giới hạn kích thước thấp hơn có thể được khuếch đại, được sử dụng trong bài báo Lipton của Williams, thực ra là do Allender và Koucký ( eccc.hpi-web.de/report/2008/038 ).
Emil Jeřábek hỗ trợ Monica

@ EmilJeřábek Cảm ơn! Tôi đã thêm giấy đó. Hy vọng rằng câu trả lời có vẻ tốt hơn bây giờ.
siuman

14

Theo đề nghị của Kaveh, tôi đưa nhận xét của mình dưới dạng câu trả lời (mở rộng).

Liên quan đến Q1 , một từ thận trọng là theo thứ tự: thậm chí logarit độ sâu nếu không hiểu rõ, không nói về đa logarit. Vì vậy, trong thế giới không đơn điệu, vấn đề thực sự ít tham vọng hơn nhiều:

Vấn đề đánh bại độ sâu log: Chứng minh giới hạn dưới siêu tuyến tính (!) Cho NC1 -circuits.

Vấn đề vẫn mở (cho bây giờ hơn 30 năm) ngay cả đối với tuyến tính -circuits. Đây là những fanin- 2 mạch trên cơ sở { , 1 } , và họ tính toán tuyến tính biến đổi f ( x ) = A x trên G F ( 2 ) . Dễ dàng đếm cho thấy hầu hết tất cả các ma trận A đều yêu cầu cổng Ω ( n 2 / log n ) , ở bất kỳ độ sâu nào. NC12{,1}f(x)=AxGF(2)AΩ(n2/logn)

Liên quan đến Q2 : Có, chúng tôi một số biện pháp đại số / tổ hợp, giới hạn dưới sẽ đánh bại các mạch sâu log. Thật không may, cho đến nay, chúng tôi không thể chứng minh giới hạn đủ lớn về các biện pháp này. Này, cho tuyến tính -circuits, một biện pháp như vậy là cứng nhắc R Một ( r ) của ma trận A . Đây là số lượng mục nhỏ nhất của A mà người ta cần thay đổi để giảm thứ hạng xuống r . Nó rất dễ dàng để hiển thị rằng R Một ( r ) (NC1 RA(r)AAr giữ cho mọima trậnboolean n × n A và Valiant (1977) đã chỉ ra rằng ràng buộc này là chặt chẽ đối với hầu hết các ma trận. Để đánh bại mạch log-chiều sâu, nó là đủ để triển lãm một chuỗi các boolean n × n ma trận Một ví dụ đóRA(r)(nr)2n×nAn×nA

cho hằng số ε , δ > 0RA(ϵn)n1+δϵ,δ>0 .

Điều tốt nhất chúng ta đã biết cho đến nay là ma trận với R Một ( r ) ( n 2 / r ) log ( n / r ) . Đối với ma trận Sylvester (tức là ma trận sản phẩm bên trong), giới hạn dưới của Ω ( n 2 / r ) rất dễ hiển thị . ARA(r)(n2/r)log(n/r)Ω(n2/r)

Chúng tôi có các biện pháp tổ hợp cho chung (non-linear) -circuits, cũng Đối với một song phương n × n đồ thị G , hãy t ( G ) là số nhỏ nhất tG có thể được viết như một giao điểm của t song phương đồ thị, mỗi là một liên minh của ít nhất t đồ thị hai phía đầy đủ. Để đánh bại các mạch có độ sâu log chung, sẽ đủ để tìm một chuỗi các biểu đồ vớiNC1n×nGt(G)tGtt

cho một hằng số ε > 0t(Gn)nϵϵ>0

(xem, ví dụ ở đây về cách điều này xảy ra). Một lần nữa, hầu hết các đồ thị có . Tuy nhiên, tốt nhất vẫn là giới hạn t ( G ) log 3 n thấp hơn cho ma trận Sylvester, dot(G)n1/2t(G)log3n Lokam .

Cuối cùng, hãy để tôi đề cập rằng chúng ta thậm chí có một phép đo tổ hợp "đơn giản" (số lượng) giới hạn dưới (tuyến tính) yếu hơn sẽ mang lại giới hạn thấp hơn theo cấp số mũ (!) Cho các mạch không đơn điệu. Đối với đồ thị lưỡng cực G , hãy để c ( G ) là số lượng nhỏ nhất của các phép toán fanin- 2 union ( ) và giao điểm ( ) cần thiết để tạo G khi bắt đầu từ các ngôi sao; một ngôi sao là một tập hợp các cạnh nối một đỉnh với tất cả các đỉnh ở phía bên kia. Hầu như tất cả các đồ thị có c ( G ) = Ω ( n 2n×nGc(G)2G . Mặt khác, giới hạn dưới củac(G)=Ω(n2/logn)

cho một hằng số ε > 0c(Gn)(4+ϵ)nϵ>0

sẽ bao hàm một ràng buộc thấp vào độ phức tạp mạch không đơn điệu của một hàm boolean rõ ràng f G của N biến. Nếu Gn × m đồ thị với m = o ( n ) , sau đó thậm chí là một ràng buộc thấp hơn c ( G n ) ( 2 + ε ) n là đủ (một lần nữa, xem, ví dụ như ở đây về cách điều này xảy ra). Giới hạn dưới c ( GΩ(2N/2)fGNGn×mm=o(n)c(Gn)(2+ϵ)n có thể được hiển thị cho các biểu đồ tương đối đơn giản. Vấn đề, tuy nhiên, là để làm điều này với " - ε " thay thế bằng " + ε ". Nhiều biện pháp kết hợp phức tạp mạch giới hạn thấp hơn (bao gồm cả A C C -circuits) có thể được tìm thấy trong cuốn sách. c(G)(2ϵ)nϵ+ϵACC

PS Vì vậy, chúng ta bởi một yếu tố liên tục của hiển thị P N P ? Dĩ nhiên là không. Tôi đã đề cập đến biện pháp sau này c ( G ) chỉ để chỉ ra rằng người ta nên xử lý "khuếch đại" (hoặc "phóng đại") các giới hạn dưới với một phần hoài nghi lành mạnh: mặc dù các giới hạn mà chúng ta cần trông "ngây thơ", nhỏ hơn nhiều ( tuyến tính) hơn hầu hết tất cả các đồ thị yêu cầu (bậc hai), khó khăn vốn có của việc chứng minh giới hạn dưới (yếu) có thể còn lớn hơn. Tất nhiên, khi tìm thấy một biện pháp kết hợp, chúng ta có thể nói2+ϵPNPc(G) điều gì đó về tính chất nào của các hàm làm cho chúng khó tính toán. Điều này có thể hữu ích để chứng minh một gián tiếpgiới hạn dưới: một số lớp phức tạp chứa một hàm yêu cầu các mạch hoặc công thức lớn. Nhưng mục tiêu cuối cùng là đưa ra một hàm cứng rõ ràng , mà định nghĩa của nó không có "mùi thuật toán", không có bất kỳ khía cạnh phức tạp tiềm ẩn nào.


2
Tôi thấy điều này rất thú vị: 1. siêu tuyến giới hạn dưới cho các hàm tuyến tính trên có vẻ là một câu hỏi giới hạn dưới rất cụ thể. 2. giới hạn dưới về các khái niệm toán học không liên quan trực tiếp đến tính toán có liên quan đến mạch giới hạn dưới. GF(2)
Kaveh

ma trận độ cứng là tuy nhiên một khái niệm rõ ràng thống nhất cấu trúc của nó dường như trái ngược mạnh mẽ với hầu hết các giới hạn thấp hơn thể hiện dưới dạng , trong khi đó là trong điều kiện thay vì Ω ( f ( n , r ) ) (hoặc nói Ω ( f ( Ω(f(n))Ω(f(n,r))trong đónlà kích thước đầu vào vì nó cho ma trận vuông). có ai nhìn thấy những cách khác để thể hiện ma trận độ cứng ví dụ vềΩ(f(n))? Ω(f(n,r))nΩ(f(n))
vzn

@vzn: Giới hạn dưới mạnh nhất trên độc lập hoặc r0 , vì R A ( n ) = 0 . Tôi sợ, bạn hiểu sai ý nghĩa của sự cứng nhắc. RA(r) r0RA(n)=0
Stasys
Khi sử dụng trang web của chúng tôi, bạn xác nhận rằng bạn đã đọc và hiểu Chính sách cookieChính sách bảo mật của chúng tôi.
Licensed under cc by-sa 3.0 with attribution required.